LSAT and Law School Admissions Forum

Get expert LSAT preparation and law school admissions advice from PowerScore Test Preparation.

User avatar
 Dave Killoran
PowerScore Staff
  • PowerScore Staff
  • Posts: 5852
  • Joined: Mar 25, 2011
|
#26436
Complete Question Explanation
(The complete setup for this game can be found here: lsat/viewtopic.php?t=11009)

The correct answer choice is (C)

If the ranks of P, O, and K were consecutive, then J’s rank would be higher than the ranks of P, O, and K, and N’s rank would be lower than the ranks of P, O, and K. The overall ranking of the seven names would be as follows:
  • 1. ..... L or J
    2. ..... J or L
    3. ..... P
    4. ..... O
    5. ..... K
    6. ..... N
    7. ..... M
Since O would be ranked fourth and N would be ranked sixth, it follows that the statement “N received more votes than O” would have to be false. Therefore, answer choice (C) is correct.
 whardy21
  • Posts: 48
  • Joined: Sep 30, 2018
|
#58943
Question: If POK is consecutive, how is J or P not #3 in the ranking. If POK is consecutive I though J has no relation to P, meaning J or P can be second or third. Please help
 James Finch
PowerScore Staff
  • PowerScore Staff
  • Posts: 943
  • Joined: Sep 06, 2017
|
#58996
Hi W. Hardy,

If P-O-K are consecutive, that means they are all together in a block, and nothing else can come between them. This pushes the J in front of the P, as the J must come before the O, and since the P is right before the O, the J must come before it as well (since we know P-O are consecutive). The same thing happens with the the N on the backend, pushing it to after the P-O-K block. This means L/J take up the first two slots, then P-O-K will go 3-5, then N 6th, and lastly M (as must always happen).

Hope this clears things up!
 lizaetillman
  • Posts: 3
  • Joined: Nov 17, 2018
|
#60619
I answered with (C).

I have a question about problems worded this way. "if....were consecutive". I thought it was a trick meaning that the three variables can be right next to each other but not exactly listed POK. They just can't have a variable in between. Like KOP or OKP or KPO..... I think I completely over thought the problem because I was focused on a "trick". :oops:
 Robert Carroll
PowerScore Staff
  • PowerScore Staff
  • Posts: 1787
  • Joined: Dec 06, 2013
|
#60704
Liza,

Unless the question stem explicitly "suspends" a rule or set of rules, all the original rules remain in effect. This question forces the three variables to be consecutive, but the original rules, which remain in effect, force them to be in a specific order as well. Hope that helps!

Robert Carroll

Get the most out of your LSAT Prep Plus subscription.

Analyze and track your performance with our Testing and Analytics Package.